Frage:
Sind diese beiden Quantensysteme unterscheidbar?
Andrew
2011-04-05 00:23:25 UTC
view on stackexchange narkive permalink

Angenommen, Stanford Research Systems beginnt mit dem Verkauf einer zweistufigen Atomfabrik. Ihr Student drückt einen Knopf und knallt, er bekommt ein zweistufiges Atom. Die Hälfte der Zeit, in der das Atom im Grundzustand produziert wird, und die Hälfte der Zeit, in der das Atom im angeregten Zustand produziert wird, aber ansonsten erhalten Sie jedes Mal genau das gleiche Atom.

National Instruments verkauft ein billiges Atom Zwei-Ebenen-Atomfabrik, die gleich aussieht, aber nicht die gleiche Leistung hat. Wenn Ihr Student in der NI-Maschine einen Knopf drückt, erhält er dasselbe zweistufige Atom wie die SRS-Maschine, aber das Atom befindet sich immer in einer 50/50-Überlagerung von Grund- und angeregten Zuständen mit einer zufälligen relativen Phase zwischen den beiden Zustände.

Die "zufällige relative Phase zwischen den beiden Zuständen" des NI-Abschlags variiert von Atom zu Atom und ist dem Benutzer des Geräts unbekannt.

Sind diese beiden Maschinen unterscheidbar? Welches Experiment würden Sie durchführen, um ihre Ausgaben zu unterscheiden?

Ich habe diese Frage gestellt, weil ich denke, dass es eine gute Folgefrage zur Ununterscheidbarkeit im QM gibt. Jedes Mal, wenn ich versuche, diese Folgefrage zu formulieren, kommt sie jedoch hoffnungslos vage und mystisch klingend heraus. Ich werde weiter umformulieren, bis ich etwas Konkretes bekomme.
Ich denke, dies ist eine großartige Frage und spielt mit einigen grundlegenden Vorstellungen davon, wie wir beschreiben, was in QM erkennbar ist und wie viel Redundanz in unserer Beschreibung enthalten ist
Ich sehe, Sie haben eine akzeptierte Antwort markiert. Ich finde diese Frage faszinierend und würde es gerne weiter diskutieren. Ich hatte wirklich gehofft, dass die Leute mehr Analysen hinzufügen würden. Da dies niemand tat, fügte ich hinzu, was auf den ersten Blick (möglicherweise falsch) ein Gegenbeispiel zu sein scheint, um hoffentlich zumindest mehr Diskussionen in Gang zu bringen.
@Edward Diese Frage wird im Wesentlichen beantwortet, aber ich möchte, dass dies auch weiter diskutiert wird, um zu klären, warum zwei nicht überlappende Abschnitte des Hilbert-Raums hier nicht unterschieden werden können. Also stellte ich eine neue Frage http://physics.stackexchange.com/questions/8123/distinguishability-in-quantum-ensembles
Sieben antworten:
Peter Shor
2011-04-05 01:04:23 UTC
view on stackexchange narkive permalink

Diese Systeme sind nicht unterscheidbar. Die durchschnittliche Dichtematrix ist dieselbe, und die Wahrscheinlichkeitsverteilung, die durch Durchführen einer Messung erhalten wird, hängt nur von der durchschnittlichen Dichtematrix ab.

Für das erste System ist die Dichtematrix $$ \ frac {1} {2 } \ left [\ left (\ begin {array} {cc} 1&0 \ cr 0&0 \ end {array} \ right) + \ left (\ begin {array} {cc} 0&0 \ cr 0&1 \ end {array} \ right ) \ right]. $$

Für das zweite System lautet die Dichtematrix $$ \ frac {1} {2 \ pi} \ int_ \ theta \ frac {1} {2} \ left ( \ begin {array} {cc} 1&e ^ {- i \ theta} \ cr e ^ {i \ theta} &1 \ end {array} \ right) d \ theta. $$

Es ist einfach überprüft, ob diese gleich sind.

Es läuft darauf hinaus, ob die Phase "wirklich zufällig" ist oder nicht, oder? Vielleicht gibt es einige experimentelle Systematiken in der billigen Nachahmung, die sich in der zufälligen relativen Phase zeigen. Wie @Peter Morgan in seiner Antwort feststellt, ist es kaum ein billiger Abschlag, wenn die Phase im Durchschnitt Null ist.
Genauer gesagt müssen wir davon ausgehen, dass die "Zufallsphase" im Ensemble gleichmäßig verteilt ist. Denn das wird implizit davon ausgegangen, wie das Integral dort geschrieben wird.
Peter Morgan
2011-04-05 01:20:31 UTC
view on stackexchange narkive permalink

Fall 1: $ \ frac {1} {2} \ left [\ left | 0 \ right> \ left<0 \ right | + \ left | 1 \ right> \ left<1 \ right | \ right] $.

Fall 2, Durchschnitt über die Phasen $ 0 $ bis $ 2 \ pi $: $$ \ frac {\ int \ left [(\ left | 0 \ right> + e ^ {i \ theta} \ left | 1 \ right>) ( \ left<0 \ right | + e ^ {- i \ theta} \ left<1 \ right |) \ right] d \ theta} {\ int \ left [(\ left<0 \ right | + e ^ {- i \ theta} \ left<1 \ right |) (\ left | 0 \ right> + e ^ {i \ theta} \ left | 1 \ right>) \ right] d \ theta}. $$ Die Kreuzterme werden auf Null gemittelt, weil $ \ int \ limit_0 ^ { 2 \ pi} e ^ {i \ theta} d \ theta = 0 $, es ist also dieselbe Dichtematrix. Wenn dies wirklich das ist, was die verschiedenen Hersteller liefern, ist es kein billiger Abschlag.

Ich habe die Nachricht "Eine andere Antwort gepostet" nur Sekunden bevor ich fertig war. Als ich sah, wie ähnlich es war, habe ich es fast nicht gepostet, aber hey, die Notation ist anders.
interessant, also physikalisch sind sie äquivalente Zustände (nach unserem Kenntnisstand), aber mathematisch in verschiedenen Formen beschrieben
@lurscher, ist wählerisch, aber ich halte fast alles an dieser Frage für mehr Mathematik als Physik. Ein Experimentator möchte, dass die Produktdatenblätter uns Aufschluss darüber geben, welche Arten von Atomen abgegeben werden, wie nahe das thermische Gleichgewicht die kinetischen Energien sind, bei welcher Temperatur, welche Atomenergieniveaus vorhanden sind, wie die Verteilung der Abgabe über die Zeit ist und vieles mehr Charakterisierung dessen, was ich bekommen würde, wenn ich die Produkte mitbringen würde. Für mich ist die Physik die Frage, ob die NI-Maschine Cäsium anstelle von Xenon liefert, eine pro Sekunde anstelle einer pro Millisekunde oder was auch immer.
@Peter: ist es? Klingt für mich nach Technik. Experimentelle Physik bestenfalls.
@Marek Sie haben Recht, dass meine Antwort auf @lurscher Probleme hat. Dennoch ist es das Ziel jeder physikalischen Theorie, genau genug, einfach genug, nachvollziehbar genug zu sein, genug von einer Vielzahl anderer Verdienste zu haben, die eine Theorie haben kann, und * in Ordnung * gut genug, um * routinemäßig * verwendet zu werden. als Engineering-Tool. Lassen Sie uns nicht darüber streiten, ob die Physik den Hund oder die Technik den Hund wedelt.
Apropos der obigen Kommentare, http://wondermark.com/634/
Kostya
2011-04-05 20:47:38 UTC
view on stackexchange narkive permalink

Lassen Sie mich einen Hinweis geben, der nützlich sein könnte, um die Dinge klarer zu machen.
Es ist Landau-Lifshitz, Buch 5, Kapitel 5:

Die Mittelung mittels der statistischen Matrix. .. hat eine zweifache Natur. Es umfasst sowohl die Mittelung aufgrund des probalistischen Charakters der Quantenbeschreibung (auch wenn diese so vollständig wie möglich ist) als auch die statistische Mittelung, die aufgrund der Unvollständigkeit unserer Informationen über das betrachtete Objekt erforderlich ist. Es muss jedoch berücksichtigt werden dass diese Bestandteile nicht getrennt werden können; Das gesamte Mittelungsverfahren wird als eine einzige Operation ausgeführt und kann nicht als Ergebnis aufeinanderfolgender Mittelungen dargestellt werden, von denen eine rein quantenmechanisch und die andere rein statistisch ist.

Diese "zweifache Mittelung" ist genau der Grund, warum die beiden Zustände in keiner Weise unterschieden werden können.

Lassen Sie mich noch ein schönes Zitat hinzufügen:

Es muss betont werden, dass die Mittelung über verschiedene $ \ psi $ -Zustände, die wir verwendet haben, um den Übergang von einer vollständigen zu einer unvollständigen quantenmechanischen Beschreibung zu veranschaulichen, haben nur eine sehr formale Bedeutung. Insbesondere wäre es völlig falsch anzunehmen, dass die Beschreibung mittels der Dichtematrix bedeutet, dass sich das Subsystem in verschiedenen $ \ psi $ -Zuständen mit verschiedenen Wahrscheinlichkeiten befinden kann und dass der Durchschnitt über diesen Wahrscheinlichkeiten liegt. Eine solche Behandlung würde im Widerspruch zu den Grundprinzipien der Quantenmechanik stehen.

Edward
2011-04-05 06:28:46 UTC
view on stackexchange narkive permalink

BEARBEITEN: Lesen Sie es einige Stunden später erneut und finden Sie meinen Fehler. Ich dachte, ich mache etwas falsch. Bei der Berechnung der bedingten Wahrscheinlichkeit habe ich Operationen außerhalb der Reihenfolge angewendet. Es ist jeweils 1/2. Ich werde den Kommentar unberührt lassen.


Ich denke, die Antwort lautet Ja, oder zumindest bin ich nicht ganz davon überzeugt, dass die Antwort Nein lautet.

Ich werde sie bereitstellen Ein Beispiel unten, aber ich finde es nicht sehr überzeugend, da ich mich nur ad-hoc ihm genähert habe und kein nettes "übergreifendes" Prinzip habe, um dies zu beseitigen. Betrachten Sie dies im Grunde eher als Kommentar, um die Diskussion in Gang zu bringen, als als eine vollständige Antwort.

Die anderen Antworten zeigen, dass der Erwartungswert für die Messung des Systems in einem bestimmten Zustand der gleiche ist. Grundsätzlich ist die Dichtematrix des Ensembles dieselbe, aber die Dichtematrix der ersten Maschine hat nur zwei mögliche Ausgänge, während die zweite eine unendliche Zahl hat. Wenn wir uns sofort auf den Ensemble-Durchschnitt konzentrieren, scheint jede Möglichkeit, sie zu unterscheiden, wegzuwerfen.


Hier ist ein Versuch, sie zu unterscheiden:

Maschine 1 mögliche Ausgabe, nur rein Zustände
$ | 0 \ rangle $
$ | 1 \ rangle $

Maschine 2 mögliche Ausgabe, beliebiger Zustand
$ \ frac {1} {\ sqrt {2}} ( | 0 \ rangle + p | 1 \ rangle) $
wobei $ p = e ^ {i \ theta} $ mit $ 0 \ le \ theta < 2 \ pi $

Nehmen Sie nun ein anderes Qubit B (es spielt hier physikalisch keine Rolle, was es ist) des vorbereiteten Zustands $ \ frac {1} {\ sqrt {2}} (| 0 \ rangle + | 1 \ rangle) $, um die Produktzustände zu erhalten:

Maschine 1
$ \ frac {1} {\ sqrt {2}} (| 0 \ rangle + | 1 \ rangle) | 0 \ rangle = \ frac {1} {\ sqrt {2}} (| 00 \ rangle + | 10 \ rangle) $
$ \ frac {1} {\ sqrt {2}} (| 0 \ rangle + | 1 \ rangle) | 1 \ rangle = \ frac {1} {\ sqrt { 2}} (| 01 \ rangle + | 11 \ rangle) $

Maschine 2
$ \ frac {1} {2} (| 0 \ rangle + | 1 \ rangle) (| 0 \ rangle + p | 1 \ rangle) = \ frac {1} {2} (| 00 \ rangle + p | 01 \ rangle + | 10 \ rangle + p | 11 \ rangle) $

Nun wollen wir eine Interaktion einführen, die einige i verursachen kann Interferenz:
$ | 00 \ rangle \ rightarrow | 00 \ rangle $
$ | 01 \ rangle \ rightarrow \ frac {1} {\ sqrt {2}} (| 01 \ rangle + | 10 \ rangle) $
$ | 10 \ rangle \ rightarrow \ frac {1} {\ sqrt { 2}} (| 01 \ rangle - | 10 \ rangle) $
$ | 11 \ rangle \ rightarrow | 11 \ rangle $

Jetzt haben wir
Maschine 1
$ \ frac {1} {\ sqrt {2}} (| 00 \ rangle + | 10 \ rangle) \ rightarrow \ frac {1} {\ sqrt {2}} | 00 \ rangle + \ frac {1} {2} ( | 01 \ rangle- | 10 \ rangle) $
$ \ frac {1} {\ sqrt {2}} (| 01 \ rangle + | 11 \ rangle) \ rightarrow \ frac {1} {\ sqrt {2 }} | 11 \ rangle + \ frac {1} {2} (| 01 \ rangle + | 10 \ rangle) $
Maschine 2
$ \ frac {1} {2} (| 00 \ rangle + p | 01 \ rangle + | 10 \ rangle + p | 11 \ rangle) \ rightarrow \ frac {1} {2} (| 00 \ rangle + p \ frac {1} {\ sqrt {2}} (| 01 \ rangle +) | 10 \ rangle) + \ frac {1} {\ sqrt {2}} (| 01 \ rangle- | 10 \ rangle) + p | 11 \ rangle) $
$ \ \ \ = \ frac {1} {2} (| 00 \ rangle + (p + 1) \ frac {1} {\ sqrt {2}} | 01 \ rangle + (p-1) \ frac {1} {\ sqrt {2}} | 10 \ rangle + p | 11 \ rangle) $

Lassen Sie uns nun zwei Messungen durchführen. Messen Sie zuerst den Zustand von B als 0 oder 1 und dann den Zustand des Atoms als 0 oder 1.

Bedingte Wahrscheinlichkeit für das Ensemble:
Wenn wir B im Zustand 1 finden, Wie groß ist die Wahrscheinlichkeit, das Atom im Zustand 0 zu finden?
Maschine 1
(1/2) x 1 + (1/2) x (1/3) = 4/6

Maschine 2
$ \ frac {\ frac {1} {2} (p-1) ^ 2} {\ frac {1} {2} (p-1) ^ 2 + p ^ 2} = \ frac { \ frac {1} {2} (2 - 2 \ cos \ theta)} {\ frac {1} {2} (2 - 2 \ cos \ theta) + 1} = \ frac {1 - \ cos \ theta} {2 - \ cos \ theta} $

Jetzt Mittelwert über $ \ theta $
$ \ mathrm {Prob} = \ frac {1} {2 \ pi} \ int_0 ^ {2 \ pi } \ frac {1 - \ cos \ theta} {2 - \ cos \ theta} d \ theta = 1 - \ frac {1} {\ sqrt {3}} $


Nun ist es gut möglich, dass ich hier einen Fehler gemacht habe. Mein Hauptpunkt ist jedoch, dass die anderen Antworten die nützlichen Informationen wegzuwerfen scheinen, um nur einen Durchschnitt der anfänglichen Ausgabezustände zu erhalten. Nach dem derzeitigen Stand der Antworten überzeugen sie mich mathematisch nicht davon, dass wir niemals einen Effekt erzielen können, indem wir Wechselwirkungen und Mehrfachmessungen mit bedingter Wahrscheinlichkeit oder möglicherweise "schwachen" Messungen hinzufügen, da die Zustände einzeln sehr unterschiedliche Dichtematrizen haben. Hoffentlich habe ich oben keinen Fehler gemacht, aber selbst wenn ich es getan hätte, würde ich immer noch gerne mehr in den anderen Antworten hören, als über das, was gerade geschrieben steht. Dies ist eine faszinierende Frage, daher bin ich sehr daran interessiert, dies weiter zu diskutieren.

+1, wenn Sie die Antwort so korrigieren, dass sie korrekt ist, anstatt einen Bearbeitungshinweis hinzuzufügen.
Interessant! Ich werde sorgfältig durchlesen.
Hmm ... es scheint das Problem zu sein, dass es ein Ensemble von Zuständen der Form $ | 0 \ rangle + e ^ {i \ theta} | 1 \ rangle $ mit * gleichmäßiger * Verteilung in Theta gab. Was Sie zu wollen scheinen, ist, mit Systemen zu interagieren und dann bedingte Anweisungen zu erfragen (oder die anderen Systeme zu verfolgen, wenn sie uns am Ende nicht interessieren), um $ | 0 \ rangle + e ^ {i \ theta} | zu erhalten 1 \ rangle \ rightarrow | 0 \ rangle + e ^ {i \ psi} | 1 \ rangle $ so, dass das Ensemble eine * ungleichmäßige * Verteilung in $ \ psi $ hat. Die Frage ist dann, können wir beweisen, dass dies unmöglich ist?
+1 Um mich zum Nachdenken zu bringen. Ich schätze auch jemanden, der seinen eigenen Fehler erkennen kann * UND * es zugeben kann.
QGR
2011-04-05 20:32:42 UTC
view on stackexchange narkive permalink

Die Dichtematrizen sind in beiden Fällen identisch. Wenn die Quantenmechanik genau linear ist, sollten beide Zustände nicht unterscheidbar sein. Wenn es jedoch einige geringfügige Nichtlinearitäten in der Zeitentwicklung gibt, sollten wir im Prinzip in der Lage sein, zwischen ihnen zu unterscheiden. Sie müssen jedoch erkennen, dass Nichtlinearitäten in der Quantenmechanik zu allen möglichen Problemen führen, weshalb die meisten Menschen davon ausgehen, dass die Quantenmechanik genau linear ist.

Marty Green
2011-08-15 02:42:21 UTC
view on stackexchange narkive permalink

Vielleicht kann die hier beschriebene Maschine tatsächlich gebaut werden. Lassen Sie mich einen beheizten Kolben vorschlagen, der ein 50-50-Gemisch aus gasförmigem, einatomigem Kohlenstoff-14 und Stickstoff-14 enthält. Wenn Sie einen Knopf drücken, öffnet sich eine Lochblende und lässt genau ein Atom entweichen. Ist es: entweder ein Kohlenstoffatom oder ein Stickstoffatom mit einer Wahrscheinlichkeit von 50% oder ist es ein Atom in einer 50-50-Überlagerung der Kohlenstoff / Stickstoff-Zustände?

EDIT: Lassen Sie uns dies tun nur ein bisschen anders. Lassen Sie uns eine Flasche mit reinem, gasförmigem monotomem Kohlenstoff-14 vorbereiten und dann 7000 Jahre warten. Lassen wir nun ein Atom aus der Flasche. Ist es ein Kohlenstoffatom, ein Stickstoffatom oder eine 50-50-Überlagerung von beiden?

Das Gewicht der Expertenmeinung in den bisher veröffentlichten Antworten scheint darauf hinzudeuten, dass diese beiden Beschreibungen experimentell sind nicht zu unterscheiden. Ich vermute, dass dies richtig ist, obwohl es eine lustige Schlussfolgerung ist, die dem gesunden Menschenverstand widerspricht, dass ein Atom entweder Kohlenstoff oder Stickstoff ist, aber nicht beides. Ich denke jedoch, dass zwei Bestimmungen beachtet werden sollten:

  1. Ich kann keinen Grund erkennen, warum ein bestimmtes Atom nicht in einer 80/20-Überlagerung erscheinen sollte, solange es langfristig ist Durchschnitt ist 50/50.

  2. Ich glaube nicht, dass die Maschine tatsächlich konstruierbar ist, weil ich nicht glaube, dass es einen Mechanismus gibt, der zuverlässig genau ein Atom gleichzeitig produzieren kann . Sie wissen nie genau, wie viele Atome Sie herausgelassen haben, und dies führt zu einer ausreichenden Unsicherheit bei der Messung, um die offensichtlichen Widersprüche zu vermeiden, die vorhanden zu sein scheinen.

  3. ol>

    EDIT: Als Andrew diese Frage stellte, versprach er eine Folgefrage. Es ist sechs Monate später und ich habe das Follow-up nicht gesehen. Ich denke, das Follow-up sollte folgendermaßen aussehen:

    Angenommen, Sie haben ein Gas im Gleichgewicht. Gemäß der Thermodynamik ist die Wahrscheinlichkeit, dass sich ein Atom in einem bestimmten Zustand befindet, durch eine Exponentialfunktion der Energie gegeben. Nach Kopenhagen haben wir also Atome in unterschiedlichen Energieeigenzuständen, die zufällige Übergänge von einem Zustand in einen anderen machen und Photonen emittieren oder absorbieren, wenn sie Übergänge machen. Frage: Gibt es eine Möglichkeit, dieses Modell experimentell von einem alternativen Modell zu unterscheiden, bei dem sich alle Atome in kontinuierlich variierenden Überlagerungen von Zuständen befinden, die kontinuierlich strahlen oder absorbieren, wenn die Ladungsverteilung dieser Überlagerungen wie winzige Antennen schwingt?

    Wenn Andrew da draußen ist, frage ich mich, ob dies seine Folgefrage war?

Bob Jaroslavski
2011-12-07 16:55:55 UTC
view on stackexchange narkive permalink

Dies ist eine so schöne philosophische Frage mit einer so ordentlichen Auflösung, dass ich nicht widerstehen kann, einen Kommentar abzugeben. Die Matrizen mit reduzierter Dichte des Atoms sind für Stanford und National gleich, aber die Quantenmechanik ist irreduzibel ganzheitlich. Die Wellenfunktion beschreibt das gesamte Universum. Wenn das Atom von Stanford hergestellt wurde, wird es auf besondere Weise mit Spuren der Umweltbilanz in Stanford verwickelt, aber wenn es bei National hergestellt wurde, wird es auf andere, aber immer noch spezifische Weise mit Spuren bei National verwickelt. Ganzheitlich gibt es tatsächlich einen Unterschied. Die Annahme, dass das Atom isoliert vom Rest der Welt betrachtet werden kann, ist ein großer Irrtum in der Quantenmechanik.



Diese Fragen und Antworten wurden automatisch aus der englischen Sprache übersetzt.Der ursprüngliche Inhalt ist auf stackexchange verfügbar. Wir danken ihm für die cc by-sa 2.0-Lizenz, unter der er vertrieben wird.
Loading...